Rédigé par Luc Giraud le . Publié dans Annales STI2D 2017.

BAC STL, STI2D Métropole 16 juin 2017

Exercice 1 6 points


Suites

La climatisation d’un véhicule automobile est un système qui a une double fonction, refroidir ou réchauffer l’habitacle. Ce système fonctionne grâce à une certaine masse de gaz réfrigérant stocké dans un réservoir.
On suppose que, par défaut d’étanchéité, le système perd naturellement 0,1 gramme de ce gaz chaque jour.
Un automobiliste possède un véhicule pour lequel la masse de gaz dans le réservoir est initialement de 660 grammes.

Partie A

Le constructeur préconise de recharger le réservoir lorsque la masse de gaz est inférieure à 440 grammes. Au bout de combien de jours le constructeur préconise-t-il à l'automobiliste de recharger ce réservoir ?

Partie B

Lors d’une visite d’entretien, le garagiste signale à l’automobiliste que le système de climatisation de son véhicule présente une baisse significative de masse de gaz : en plus de la perte naturelle de 0,1 gramme, le système perd 1 % de sa masse de gaz chaque jour.
Le garagiste recharge alors complètement le réservoir.
Pour tout entier naturel $n$, on note $u_n$ la masse de gaz dans le réservoir au bout de $n$ jours après cette visite.
On a donc $u_0 = 660$ et on admet que pour tout entier naturel $n$, on a : $u_{n+1} = 0,99u_n -0,1$.

  1. Calculer $u_1$ et $u_2$.
  2. Voici un algorithme qui, lorsque l’on saisit un nombre N non nul de jours écoulés, calcule et affiche la masse de gaz restant dans le système. $$ \begin{array}{|r l|}\hline &\text{Variables}\\ &\hspace{0.6cm} N: \text{ un nombre entier naturel}\\ &\hspace{0.6cm} k : \text{ un nombre entier naturel}\\ &\hspace{0.6cm} u : \text{ un nombre réel }\\ &\text{Entrée}\\ &\hspace{0.6cm} Saisir N\\ &\text{Initialisation}\\ &\hspace{0.6cm} u \text{ prend la valeur } 660\\ &\text{Traitement}\\ &\hspace{0.6cm} \text{ Pour } k \text{ allant de 1 à } \cdots \\ &\hspace{1cm} u \text{ prend la valeur } \cdots \\ &\hspace{0.6cm} \text{ Fin Pour} \\ &\text{Sortie}\\ &\hspace{0.6cm} \text{ Afficher } u \\ \hline \end{array} $$
    1. Recopier et compléter la partie relative au traitement de cet algorithme.
    2. Quelle masse de gaz restera-t-il au bout de 20 jours ? Arrondir au gramme près.
  3. Soit la suite $\left( y_n\right) $ définie pour tout entier naturel $n$ par $v_n=u_n + 10$.
    1. Calculer $v_0$.
    2. On admet que $\left( y_n\right) $ est une suite géométrique de raison 0,99.
      Pour tout entier naturel $n$, exprimer $v_n$ en fonction de $n$.
    3. En déduire que, pour tout entier naturel $n$, on a : $u_n =670 \times 0,99^n -10$.
    4. À l’aide de cette expression, vérifier le résultat obtenu à la question 2.b.
  4. On rappelle que le constructeur préconise de recharger le réservoir au plus tard lorsque la masse de gaz est inférieure à 440 g.
    Le coût d’une recharge est de 80 euros. Le garagiste propose de réparer le système pour 400 euros.
    Pourquoi est-il plus économique pour cet automobiliste de réparer le système? Justifier la réponse.

Correction de l'exercice 1 (6 points)

 


Suites

La climatisation d’un véhicule automobile est un système qui a une double fonction, refroidir ou réchauffer l’habitacle. Ce système fonctionne grâce à une certaine masse de gaz réfrigérant stocké dans un réservoir.
On suppose que, par défaut d’étanchéité, le système perd naturellement 0,1 gramme de ce gaz chaque jour.
Un automobiliste possède un véhicule pour lequel la masse de gaz dans le réservoir est initialement de 660 grammes.

Partie A

Le constructeur préconise de recharger le réservoir lorsque la masse de gaz est inférieure à 440 grammes. Au bout de combien de jours le constructeur préconise-t-il à l'automobiliste de recharger ce réservoir ?

Initialement, il y a 660 grammes de gaz dans le réservoir, il devra recharger ce réservoir lorsque la masse de gaz est inférieure à 440 grammes. Il se sera échappé 220 grammes de gaz, ( 660-440= 220). Ces 220 grammes se seront échappés en 2200 jours. ( 2200/0,1= 2200).
L'automobiliste devra donc recharger le réservoir au bout de 2200 jours( 6 ans et 10 jours).

Partie B

Lors d’une visite d’entretien, le garagiste signale à l’automobiliste que le système de climatisation de son véhicule présente une baisse significative de masse de gaz : en plus de la perte naturelle de 0,1 gramme, le système perd 1 % de sa masse de gaz chaque jour.
Le garagiste recharge alors complètement le réservoir.
Pour tout entier naturel $n$, on note $u_n$ la masse de gaz dans le réservoir au bout de $n$ jours après cette visite.
On a donc $u_0 = 660$ et on admet que pour tout entier naturel $n$, on a : $u_{n+1} = 0,99u_n -0,1$.

  1. Calculer $u_1$ et $u_2$.
  2. $$\begin{array}{rl} u_{ 1} &= 0,99u_0 -0,1 \\ & = 0,99\times 660 -0,1\\ &= 653,3 \end{array}$$ $$\begin{array}{rl} u_{ 2} &= 0,99u_1 -0,1 \\ & = 0,99\times 653,3 -0,1\\ &\approx 646,7 \text{à 0,1 près} \end{array}$$ $u_{ 1}=653,3$ et $u_{ 2}\approx 646,7 \text{à 0,1 près} $
  3. Voici un algorithme qui, lorsque l’on saisit un nombre N non nul de jours écoulés, calcule et affiche la masse de gaz restant dans le système. $$ \begin{array}{|r l|}\hline &\text{Variables}\\ &\hspace{0.6cm} N: \text{ un nombre entier naturel}\\ &\hspace{0.6cm} k : \text{ un nombre entier naturel}\\ &\hspace{0.6cm} u : \text{ un nombre réel }\\ &\text{Entrée}\\ &\hspace{0.6cm} \text{ Saisir } N\\ &\text{Initialisation}\\ &\hspace{0.6cm} u \text{ prend la valeur } 660\\ &\text{Traitement}\\ &\hspace{0.6cm} \text{ Pour } k \text{ allant de 1 à }\cdots\\ &\hspace{1cm} u \text{ prend la valeur } \cdots \\ &\hspace{0.6cm} \text{ Fin Pour} \\ &\text{Sortie}\\ &\hspace{0.6cm} \text{ Afficher } u \\ \hline \end{array} $$
    1. Recopier et compléter la partie relative au traitement de cet algorithme.
    2. $$\begin{array}{rl} u_{ 1} &= 0,99u_0 -0,1 \\ & = 0,99\times 660 -0,1\\ &= 653,3 \end{array}$$ $$\begin{array}{rl} u_{ 2} &= 0,99u_1 -0,1 \\ & = 0,99\times 653,3 -0,1\\ &\approx 646,7 \text{à 0,1 près} \end{array}$$ $u_{ 1}=653,3$ et $u_{ 2}\approx 646,7 \text{à 0,1 près} $
    3. Voici un algorithme qui, lorsque l’on saisit un nombre N non nul de jours écoulés, calcule et affiche la masse de gaz restant dans le système. $$ \begin{array}{|r l|}\hline &\text{Variables}\\ &\hspace{0.6cm} N: \text{ un nombre entier naturel}\\ &\hspace{0.6cm} k : \text{ un nombre entier naturel}\\ &\hspace{0.6cm} u : \text{ un nombre réel }\\ &\text{Entrée}\\ &\hspace{0.6cm} \text{ Saisir } N\\ &\text{Initialisation}\\ &\hspace{0.6cm} u \text{ prend la valeur } 660\\ &\text{Traitement}\\ &\hspace{0.6cm} \text{ Pour } k \text{ allant de 1 à }\color{red}{N }\\ &\hspace{1cm} u \text{ prend la valeur } \color{red}{0.99 u -0.1 }\\ &\hspace{0.6cm} \text{ Fin Pour} \\ &\text{Sortie}\\ &\hspace{0.6cm} \text{ Afficher } u \\ \hline \end{array} $$
    4. Quelle masse de gaz restera-t-il au bout de 20 jours ? Arrondir au gramme près.
    5. Une méthode consiste à programmer la suite $\left( u_n\right) $
      Il restera environ 538 grammes de gaz au bout de 20 jours.
  4. Soit la suite $\left( y_n\right) $ définie pour tout entier naturel $n$ par $v_n=u_n + 10$.
    1. Calculer $v_0$.
    2. $v_0=u_0+10= 660 +10= 670$. $v_0=670$
    3. On admet que $\left( y_n\right) $ est une suite géométrique de raison 0,99.
      Pour tout entier naturel $n$, exprimer $v_n$ en fonction de $n$.
    4. Comme $\left( v_n\right) $ est une suite géométrique de raison 0,99, on déduit $v_n = q^n \times v_0$ $$\begin{array}{rl} v_n &= q^n \times v_0 \\ & =0,99^n \times 670\\ & \end{array}$$ $v_n =670\times 0,99^n$.
    5. En déduire que, pour tout entier naturel $n$, on a : $u_n =670 \times 0,99^n -10$.
    6. Ayant $v_n=u_n + 10$, on déduit $u_n=v_n-10= 670 \times 0,99^n -10$
    7. À l’aide de cette expression, vérifier le résultat obtenu à la question 2.b.
    8. On calcule $$\begin{array}{rl} u_{20}&= 670 \times 0,99^{20} -10 \\ & \approx538 \\ \end{array}$$ $u_{20}\approx 538$
  5. On rappelle que le constructeur préconise de recharger le réservoir au plus tard lorsque la masse de gaz est inférieure à 440 g.
    Le coût d’une recharge est de 80 euros. Le garagiste propose de réparer le système pour 400 euros.
    Pourquoi est-il plus économique pour cet automobiliste de réparer le système? Justifier la réponse.
  6. Déterminons au bout de combien de jours il faudrait recharger le réservoir.
    On résout $u_n < 440$ $$\begin{array}{rll} u_n < 440& \iff 670 \times 0,99^{n} -10 < 440& \\ & \iff 670 \times 0,99^{n} < 450 & \\ & \iff 0,99^{n} <\dfrac{450}{ 670} &\text{ car } 670 > 0 \\ &\iff \ln \left( 0,99^{n} \right) < \ln \left( \dfrac{450}{ 670} \right) &\text{ car } \ln \text{ est strictement croissante sur } ]0;+\infty[ \\ &\iff n\ln \left( 0,99 \right) < \ln \left( \dfrac{450}{ 670} \right) &\text{ car } \ln\left( a^n\right) =n \ln a \\ &\iff n > \dfrac{\ln \left( \dfrac{450}{ 670} \right)}{\ln \left( 0,99 \right)} &\text{ car } 0,99 < 1 \text{ donc } \ln 0,99 < 0 \\ \end{array}$$ Or $ \dfrac{\ln \left( \dfrac{450}{ 670} \right)}{\ln \left( 0,99 \right)}\approx 39,6$.
    Donc dans ces conditions, il faudrait utiliser une recharge de 80 euros tous les 40 jours, on dépassera donc le montant de la réparation au bout de $ 5\times 40 = 200$ jours. $\dfrac{400}{80}= 5$
    Au bout de 200 jours, les 5 recharges utilisées compensent le montant de la réparation, et donc il est plus économique pour cet automobiliste de réparer le système.

Exercice 2 5 points


Equations différentielles

La fonte GS (graphite sphéroïdal) possède des caractéristiques mécaniques élevées et proches de celles des aciers. Une entreprise fabrique des pièces de fonte GS qui sont utilisées dans l’industrie automobile.
Ces pièces sont coulées dans des moules de sable et ont une température de 1400 ℃ à la sortie du four. Elles sont entreposées dans un local dont la température ambiante est maintenue à une température de 30 ℃ . Ces pièces peuvent être démoulées dès lors que leur température est inférieure à 650 ℃ .
La température en degrés Celsius d’une pièce de fonte est une fonction du temps $t$, exprimé en heures, depuis sa sortie du four. On admet que cette fonction $f$, définie et dérivable sur l’intervalle $[0 ; +\infty[$, est une solution sur cet intervalle de l’équation différentielle $y' + 0,065y = 1,95$.

    1. Résoudre sur $[0 ; +\infty[$ l’équation différentielle $y' + 0,065y = 1,95$.
    2. Donner$f(0)$ et vérifier que la fonction $f$ est définie sur l’intervalle $[0 ; +\infty[$ par $f(t)= 1370\text{e}^{-0,065t} + 30$.
    1. Étudier mathématiquement le sens de variation de la fonction $f$ sur l’intervalle $[0 ; +\infty[$.
    2. Pourquoi ce résultat était-il prévisible ?
  1. La pièce de fonte peut-elle être démoulée après avoir été entreposée 5 heures dans le local?
    1. Déterminer au bout de combien de temps au minimum la pièce pourra être dé moulée. Arrondir le résultat à la minute près.
    2. Pour éviter la fragilisation de la fonte, il est préférable de ne pas démouler la pièce avant que sa température ait atteint 325 ℃.
      Dans ce cas, faudra-t-il attendre exactement deux fois plus de temps que pour un démoulage à 650 ℃ ? Justifier la réponse.

Correction de l'exercice 2 (5 points)


Equations différentielles

La fonte GS (graphite sphéroïdal) possède des caractéristiques mécaniques élevées et proches de celles des aciers. Une entreprise fabrique des pièces de fonte GS qui sont utilisées dans l’industrie automobile.
Ces pièces sont coulées dans des moules de sable et ont une température de 1400 ℃ à la sortie du four. Elles sont entreposées dans un local dont la température ambiante est maintenue à une température de 30 ℃ . Ces pièces peuvent être démoulées dès lors que leur température est inférieure à 650 ℃ .
La température en degrés Celsius d’une pièce de fonte est une fonction du temps $t$, exprimé en heures, depuis sa sortie du four. On admet que cette fonction $f$, définie et dérivable sur l’intervalle $[0 ; +\infty[$, est une solution sur cet intervalle de l’équation différentielle $y' + 0,065y = 1,95$.

    1. Résoudre sur $[0 ; +\infty[$ l’équation différentielle $y' + 0,065y = 1,95$.
    2. L'équation différentielle $y' + 0,065y =1,95$ s'écrit $y'= -0,065y +1,95$.\\ Cette équation est donc du type $y'= ay +b$, où $a= -0,065$ et $b=1,9$.
      La solution générale de l'équation $y'= ay +b$ est $y= -\frac{b}{a}+C\text{e}^{at}$.
      Ici $-\frac{b}{a}= -\frac{1,95}{-0,065}=30$.
      La solution générale de l'équation est$y= 30+C\text{e}^{ -0,065t}$ où $C$ désigne une constante réelle.
    3. Donner$f(0)$ et vérifier que la fonction $f$ est définie sur l’intervalle $[0 ; +\infty[$ par $f(t)= 1370\text{e}^{-0,065t} + 30$.
    4. Les pièces sont à la température de 1400℃. Donc $f(0)= 1400$.
      Comme $f$ est une solution de léquation différentielle, on a $f(t)= 30+C\text{e}^{ -0,065t}$ où $C$ désigne une constante réelle. $$\begin{array}{rl} f(0)=1400& \iff 30+C\text{e}^{ -0,065\times 0 }=1400\\ & \iff 30+C\text{e}^{ 0 }=1400\\ & \iff 30+C =1400\\ & \iff C =1370\\ \end{array}$$ La fonction $f$ est donc définie sur l’intervalle $[0 ; +\infty[$ par $f(t)= 30+1370\text{e}^{ -0,065t}$
    1. Étudier mathématiquement le sens de variation de la fonction $f$ sur l’intervalle $[0 ; +\infty[$.
    2. On calcule la dérivée de $f$.
      $f(t)= 30+1370\text{e}^{ -0,065t}$ et $\left( e^u \right)' =u' e^u $ donc : $$\begin{array}{rl} f'(t)& = 1370\times \left( -0,065\right)\text{e}^{ -0,065t} \\ & = -89,05 \text{e}^{ -0,065t}\\ & \end{array}$$ La fonction exponentielle étant strictement positive sur $\mathbb R$ , on déduit que pour tout $t\in [0 ; +\infty[$ on a $ \text{e}^{ -0,065t}> 0$ et donc $-89,05 \text{e}^{ -0,065t}< 0$.
      La dérivée étant strictement négative sur l’intervalle $[0 ; +\infty[$, la fonction $f$ est strictement décroissante sur l’intervalle $[0 ; +\infty[$.
    3. Pourquoi ce résultat était-il prévisible ?
    4. Comme la température ambiante est de 30 ℃, et comme les pièces sont à la température de 1400 ℃, la température des pièces va diminuer et donc $f$ est bien une fonction décroissante du temps.
  1. La pièce de fonte peut-elle être démoulée après avoir été entreposée 5 heures dans le local?
  2. Calculons $f(5) = 30+C\text{e}^{ -0,065\times 5 }\approx 1020$
    Au bout de 5 heures,la température des piéces est de 1200 ℃ environ et donc les pièces ne peuvent pas être démoulées.
    1. Déterminer au bout de combien de temps au minimum la pièce pourra être dé moulée. Arrondir le résultat à la minute près.
    2. On résout l'équation $f(t)< 650$ $$\begin{array}{rll} f(t)< 650 & \iff 30+1370\text{e}^{ -0,065t} < 650& \\ & \iff 1370\text{e}^{ -0,065t} < 620&\\ & \iff \text{e}^{ -0,065t} < \dfrac{620}{1370} & \text{ car } 1370> 0\\ & \iff \ln \left( \text{e}^{ -0,065t}\right) < \ln \left( \dfrac{62}{137}\right) & \ln \text{ est strictement croissante sur } ]0;+\infty[ \\ & \iff -0,065t < \ln \left( \dfrac{62}{137}\right) & \\ & \iff t > \dfrac{\ln \left( \dfrac{62}{137}\right)}{-0,065} & \text{ car } -0,065 < 0 \\ \end{array}$$ Or $ \dfrac{\ln \left( \dfrac{62}{137}\right)}{-0,065}\approx 12,198 \; h$ soit environ 12 heures et 12 minutes.
      Les pièces pourront êtree démoulées au bout de 12 heures et 12 minutes.
    3. Pour éviter la fragilisation de la fonte, il est préférable de ne pas démouler la pièce avant que sa température ait atteint 325 ℃.
      Dans ce cas, faudra-t-il attendre exactement deux fois plus de temps que pour un démoulage à 650 ℃ ? Justifier la réponse.
    4. On résout de même $f(t)<325$ $$\begin{array}{rll} f(t)< 650 & \iff 30+1370\text{e}^{ -0,065t} < 325& \\ & \iff 1370\text{e}^{ -0,065t} < 295&\\ & \iff \text{e}^{ -0,065t} < \dfrac{295}{1370} & \text{ car } 1370> 0\\ & \iff \ln \left( \text{e}^{ -0,065t}\right) < \ln \left( \dfrac{59}{274}\right) & \ln \text{ est strictement croissante sur } ]0;+\infty[ \\ & \iff -0,065t < \ln \left( \dfrac{59}{274}\right) & \\ & \iff t > \dfrac{\ln \left( \dfrac{59}{274}\right)}{-0,065} & \text{ car } -0,065 < 0 \\ \end{array}$$ Or $ \dfrac{\ln \left( \dfrac{59}{274}\right)}{-0,065}\approx 23,624 \; h$ soit environ 23 heures et 38 minutes.
      Il est donc faux qu'il faut attendre exactement deux fois plus de temps que pour un démoulage à 650 ℃ .

Exercice 3 4 points


Probabilités

Un chef cuisinier décide d’ajouter un « menu terroir » à la carte de son restaurant. S’appuyant sur sa longue expérience, le restaurateur pense qu’environ 30% des clients choisiront ce menu. Ceci le conduit à faire l’hypothèse que la probabilité qu’un client, pris au hasard, commande le « menu terroir » est $ p = 0,3$.

Partie A

Afin de tester la validité de son hypothèse, le restaurateur choisit au hasard 100 clients et observe que 26 d’entre eux ont commandé un « menu terroir » .
Après discussion avec son comptable, le restaurateur décide d’accepter l’hypothèse que $p = 0,3$.
À l’aide d’un intervalle de fluctuation asymptotique à 95%, justifier cette décision.

Partie B

Une agence de voyage a réservé toutes les tables du restaurant pour la semaine à venir. Le restaurateur sait ainsi que 1000 clients viendront déjeuner chacun une fois durant la semaine. Le nombre de « menus terroir » qui seront alors commandés est une variable aléatoire $X$.
On considère que la probabilité qu’un des clients commande un « menu terroir « est $p = 0,3$.

  1. On admet que la variable aléatoire $X$ suit une loi binomiale.
    1. Donner ses paramètres.
    2. Déterminer la probabilité que le nombre de « menu terroir « commandés soit inférieur ou égal à 315.
  2. On décide d’approcher la loi binomiale précédente par la loi normale d'espérance $\mu = 300$ et d’écart type $\sigma = 14,49$.
    Justifier les valeurs de $\mu$ et $\sigma$.
    Dans la suite de l’exercice, on utilisera cette approximation par la loi normale. Les résultats seront arrondis à $10^{-2}$ près.
    1. Estimer $P(285 \leqslant X \leqslant315)$.
    2. Estimer $P( X\geqslant 350) $ et interpréter le résultat obtenu.

Correction de l'exercice 3 (4 points)


Probabilités

Un chef cuisinier décide d’ajouter un « menu terroir » à la carte de son restaurant. S’appuyant sur sa longue expérience, le restaurateur pense qu’environ 30% des clients choisiront ce menu. Ceci le conduit à faire l’hypothèse que la probabilité qu’un client, pris au hasard, commande le « menu terroir » est $ p = 0,3$.

Partie A

Afin de tester la validité de son hypothèse, le restaurateur choisit au hasard 100 clients et observe que 26 d’entre eux ont commandé un « menu terroir » .
Après discussion avec son comptable, le restaurateur décide d’accepter l’hypothèse que $p = 0,3$.
À l’aide d’un intervalle de fluctuation asymptotique à 95%, justifier cette décision.

L'intervalle de fluctuation asymptotique au seuil de 95 % est : $$I= \left[ p - 1,96 \dfrac{\sqrt{p(1-p)}}{\sqrt{n}} \,;\, p + 1,96 \dfrac{\sqrt{p(1-p)}}{\sqrt{n}}\right] $$ $$\begin{array}{rl} I_{100} &= \left[0,3- 1,96\sqrt{\dfrac{0,3\times 0,7}{100}}~;~0,3 + 1,96\sqrt{\dfrac{0,3\times 0,7}{100}} \right] \\ &= \left[ 0,3 - 1,96 \dfrac{\displaystyle\sqrt{0,21}}{\sqrt{100}} \,;\, 0,3 + 1,96 \dfrac{\displaystyle\sqrt{0,21}}{\sqrt{100}} \right] \\ \end{array}$$

$$I_{100}\approx[ 0,210; 0,390]$$ L'intervalle de fluctuation asymptotique au seuil de 95 % est : $I_{100} \approx \left[ 0,210; 0,390 \right]$
La fréquence observée est $f_{\text{Obs}}=\dfrac{26 }{100}=0,26$.
Comme $f_{\text{Obs}}\in I_{100}$. On accepte l’hypothèse que $p = 0,3$.

Partie B

Une agence de voyage a réservé toutes les tables du restaurant pour la semaine à venir. Le restaurateur sait ainsi que 1000 clients viendront déjeuner chacun une fois durant la semaine. Le nombre de « menus terroir » qui seront alors commandés est une variable aléatoire $X$.
On considère que la probabilité qu’un des clients commande un « menu terroir « est $p = 0,3$.

  1. On admet que la variable aléatoire $X$ suit une loi binomiale.
    1. Donner ses paramètres.
    2. On répète $\1$  fois, de façon indépendante, l’expérience «\2 » qui comporte 2 issues :

      • « \3 » considéré comme succès, de probabilité $p=\4$
      • « \5 » considéré comme échec, de probabilité $q=1-p=\6$

      Nous sommes donc en présence d’un schéma de Bernoulli et la variable aléatoire $\7$ prenant pour valeurs le nombre de succès obtenus suit la loi binomiale de paramètres $\1$  et $\4$ notée $\mathscr{B}(\1;\4)$ .

      Pour tout entier $k$ où $0\leq k\leq \1$, on a $$P(\7=k)=\binom{\1}{k}\times \left(\4\right)^k\times\left( \6\right)^{\1-k}$$

    3. Déterminer la probabilité que le nombre de « menu terroir « commandés soit inférieur ou égal à 315.
    4.  

      2ND DISTR AbinomFRép( \1 , \2,\3)EXE
      Avec une calculatrice de type TI $$binomFR\text{é}p(\1,\2,\3) \approx \4$$

      $$P( \5 \leq \3)\approx \4 \text{ à } 10^{-\6} \text{ près.}$$
  2. On décide d’approcher la loi binomiale précédente par la loi normale d'espérance $\mu = 300$ et d’écart type $\sigma = 14,49$.
    Justifier les valeurs de $\mu$ et $\sigma$.
  3. On approche $X$ par une loi normale qui a les mêmes paramètres.
    Son espérance est donc $\mu =E(X)=np= 1000\times 0,3= 300$.
    Son écart-type est $\sigma =\sqrt{npq}=\sqrt{300\times 0,7}=\sqrt{210}\approx 14,49$. Il est donc légitime d'approcher la loi binomiale $X$ par la loi normale d'espérance $\mu = 300$ et d’écart type $\sigma = 14,49$.
    Dans la suite de l’exercice, on utilisera cette approximation par la loi normale. Les résultats seront arrondis à $10^{-2}$ près.
    1. Estimer $P(285 \leqslant X \leqslant315)$.
    2. Notons $Y$ la loi $\mathcal{N}(270,14,49)$ $$\begin{array}{rl} P(285 \leqslant X \leqslant315) & \approx P(285 \leqslant Y \leqslant315) \\ \end{array}$$

      2ND DISTR 2NORMALFRép( \1 , \2,\3,\4)EXE
      Avec une calculatrice de type TI

      $$NormalFR\text{é}p(\1,\2,\3,\4) \approx \5$$

      $$P(\1 \leq \6 \leq \2)\approx \5 \text{ à } 10^{-\7} \text{ près.}$$

       

      $P(285 \leqslant X \leqslant315)\approx 0.699$
    3. Estimer $P( X\geqslant 350) $ et interpréter le résultat obtenu.
    4. $$\begin{array}{rl} P( X\geqslant 350)&\approx P( Y\geqslant 350) \\ & \approx \\ &\text{normalFRép}(350,10^{99},300,14.49)\\ &\approx 2.8\times 10^{-4} \end{array}$$

       

      2ND DISTR 2NORMALFRép( $\1$ , $10^{99}$,\2,$\3$)EXE
      Avec une calculatrice de type TI

      $$NormalFR\text{é}p(\1,10^{99},\2,\3) \approx \4$$

      $$P( \5 \geq \1)\approx \4 \text{ à } 10^{-\6} \text{ près.}$$
      $P( X \geqslant350)\approx 2.8\times 10^{-4}$. On est presque sûr que moins de 350 clients choisiront le menu terrroir.

Exercice 4 4 points


QCM Nombres complexes, loi exponentielle , optimisation

Pour chacune des propositions suivantes, indiquer si elle est vraie ou fausse et justifier la réponse choisie. Toute trace de recherche, même incomplète ou non fructueuse, sera prise en compte dans l’évaluation.
Une réponse non justifiée ne rapporte aucun point.

  1. Proposition 1 : Le nombre complexe $z$ de module $4\sqrt 3$ et dont un argument est$\frac{2\pi}{3}$ a pour forme algébrique $-2\sqrt 3 + 6i$.
  2. Le plan est muni d’un repère orthonormé direct $\left(\text{O},~\vec{u},~\vec{v}\right)$. Les points A, B et C ont pour affixes respectives $z_A=2\text{e}^{i\frac{\pi}{2}} , z_B = -1 + i\sqrt 3$ et $z_C = z_A\times z_B$.
    Proposition 2 : Le point $C$ appartient au cercle de centre O et de rayon 4.
  3. On a tracé ci-dessous dans un repère orthonormé $\left(\text{O},~\vec{i},~\vec{j}\right)$ la courbe représentative de la fonction $f$ définie sur l’intervalle [0 ; 2] par $f(x) = -\frac{1}{2}x+ 1$.
    On considère un point M de coordonnées $ \left( x, -\frac{1}{2}x+ 1\right) $ sur la courbe $\mathcal{C}$, ainsi que les points $H(x,0)$ et $K(0,-\frac{1}{2}x+ 1)$.

    Proposition 3 : L'aire, en unités d’aire, du rectangle $OHMK$ est maximale lorsque $M$ a pour abscisse 1.
  4. On peut modéliser le temps d’attente d’un client, en minutes, à la caisse d’un supermarché par une variable aléatoire $T$ qui suit une loi exponentielle de paramètre $\lambda$.
    Des études statistiques montrent que la probabilité qu’un client attende plus de 7 minutes à cette caisse est 0,417.
    On rappelle que pour tout réel $t$ positif, $P(T> t) =\text{e}^{-\lambda t}$.
    Proposition 4 : Le temps moyen d’attente à cette caisse de supermarché est 9 minutes.

 


Exercice 4 4 points


QCM Nombres complexes, loi exponentielle , optimisation

Pour chacune des propositions suivantes, indiquer si elle est vraie ou fausse et justifier la réponse choisie. Toute trace de recherche, même incomplète ou non fructueuse, sera prise en compte dans l’évaluation.
Une réponse non justifiée ne rapporte aucun point.

  1. Proposition 1 : Le nombre complexe $z$ de module $4\sqrt 3$ et dont un argument est$\frac{2\pi}{3}$ a pour forme algébrique $-2\sqrt 3 + 6i$.
  2. Le nombre complexe $z$ de module $4\sqrt 3$ et dont un argument est$\frac{2\pi}{3}$ a pour forme algébrique $-2\sqrt 3 + 6i$.\\ Le nombre complexe $z$ de module $4\sqrt 3$ et dont un argument est$\frac{2\pi}{3}$ s'écrit : $$\begin{array}{rl} z &=4\sqrt 3 \text{e}^{i \frac{2\pi}{3}} \\ &= 4\sqrt 3 \left( \cos \left( \frac{2\pi}{3}\right) +i \sin \left( \frac{2\pi}{3}\right) \right) \\ &= 4\sqrt 3 \left( -\frac{1}{2}+i \frac{\sqrt 3}{2} \right) \\ &= 4\sqrt 3 \times \left( -\frac{1}{2}\right) + 4\sqrt 3 \times \frac{\sqrt 3}{2}\\ &= -2\sqrt 3 + 6i \end{array}$$
    La proposition 1 est donc vraie.
  3. Le plan est muni d’un repère orthonormé direct $\left(\text{O},~\vec{u},~\vec{v}\right)$. Les points A, B et C ont pour affixes respectives $z_A=2\text{e}^{i\frac{\pi}{2}} , z_B = -1 + i\sqrt 3$ et $z_C = z_A\times z_B$.
    Proposition 2 : Le point $C$ appartient au cercle de centre O et de rayon 4.
  4. On peut par exemple calculer $z_C$ sous forme algébrique puis calculer son module. $$\begin{array}{rl} z_A=2\text{e}^{i\frac{\pi}{2}}&= 2 \left( \cos \left( \frac{\pi}{2}\right) +i \sin \left( \frac{\pi}{2}\right) \right) \\ & = 2( 0 + i)\\ &= 2i \end{array}$$ On a alors : $$\begin{array}{rl} z_C = z_A\times z_B& = 2i \times \left( -1 + i\sqrt 3\right) \\ & = -2i +2 i^2\sqrt 3\\ &= -2\sqrt 3 -2i \end{array}$$ On déduit alors : $$\begin{array}{rl} OC = |z_C |&=\sqrt{a^2+b^2} \\ & =\sqrt{ (2\sqrt 3)^2+2^2}\\ &=\sqrt{ 12+4}=\sqrt{16}\\ &= 4 \\ \end{array}$$ $OC=4$; donc le point $C$ appartient au cercle de centre O et de rayon 4.
    La proposition 2 est donc vraie.
  5. On a tracé ci-dessous dans un repère orthonormé $\left(\text{O},~\vec{i},~\vec{j}\right)$ la courbe représentative de la fonction $f$ définie sur l’intervalle [0 ; 2] par $f(x) = -\frac{1}{2}x+ 1$.
    On considère un point M de coordonnées $ \left( x, -\frac{1}{2}x+ 1\right) $ sur la courbe $\mathcal{C}$, ainsi que les points $H(x,0)$ et $K(0,-\frac{1}{2}x+ 1)$.


    Proposition 3 : L'aire, en unités d’aire, du rectangle $OHMK$ est maximale lorsque $M$ a pour abscisse 1.
  6. L'aire du rectangle $OHMK$ est lafonction $g$ définie sur $g$ définie sur $[0;2]$ par $ g(x)= OH\times OK$.
    Or $OH= x$ et $OK= -\frac{1}{2}x+ 1$.
    On a donc $g(x)= x\left( -\frac{1}{2}x+ 1\right) =-\frac{1}{2}x^2+ x$.
    Calculons sa dérivée :
    $g'(x)= -\frac{1}{2}\times 2x+1 =-x+1$
    Etudions le signe de la dérivée :
    $g'(x)> 0 \iff 1-x> 0 \iff -x> -1\iff x<1$ On déduit le tableau de variations de $g$ sur $[0;2]$:

    Ainsi l'aire, en unités d’aire, du rectangle $OHMK$ est maximale lorsque $M$ a pour abscisse 1.
    La proposition 3 est donc vraie.
  7. On peut modéliser le temps d’attente d’un client, en minutes, à la caisse d’un supermarché par une variable aléatoire $T$ qui suit une loi exponentielle de paramètre $\lambda$.
    Des études statistiques montrent que la probabilité qu’un client attende plus de 7 minutes à cette caisse est 0,417.
    On rappelle que pour tout réel $t$ positif, $P(T> t) =\text{e}^{-\lambda t}$.
    Proposition 4 : Le temps moyen d’attente à cette caisse de supermarché est 9 minutes.
  8. D'après l'énoncé $P(T> 7) =0,417$. $$\begin{array}{rl} P(T> 7) =0,417 & \iff \text{e}^{-7\lambda} =0,417\\ &\iff \ln \left( \text{e}^{-7\lambda}\right) =\ln(0,417)\\ &\iff -7 \lambda =\ln(0,417) \\ &\iff \lambda = -\dfrac{ \ln(0,417)}{7} \end{array}$$ Le temps d'attente moyen à ce supermarché est $E(T)= \dfrac{1}{\lambda }= -\dfrac{7}{ \ln(0,417)}\approx 8,003$
    Le temps d'attente moyen à ce supermarché est envion 8 miutes.
    La proposition 4 est donc fausse.